Last visit was: 20 Jun 2025, 16:12 It is currently 20 Jun 2025, 16:12
Close
GMAT Club Daily Prep
Thank you for using the timer - this advanced tool can estimate your performance and suggest more practice questions. We have subscribed you to Daily Prep Questions via email.

Customized
for You

we will pick new questions that match your level based on your Timer History

Track
Your Progress

every week, we’ll send you an estimated GMAT score based on your performance

Practice
Pays

we will pick new questions that match your level based on your Timer History
Not interested in getting valuable practice questions and articles delivered to your email? No problem, unsubscribe here.
Close
Request Expert Reply
Confirm Cancel
User avatar
EMPOWERgmatVerbal
User avatar
EMPOWERgmat Instructor
Joined: 23 Feb 2015
Last visit: 17 Feb 2025
Posts: 1,694
Own Kudos:
15,030
 [20]
Given Kudos: 766
Expert
Expert reply
Posts: 1,694
Kudos: 15,030
 [20]
3
Kudos
Add Kudos
17
Bookmarks
Bookmark this Post
Most Helpful Reply
User avatar
EMPOWERgmatVerbal
User avatar
EMPOWERgmat Instructor
Joined: 23 Feb 2015
Last visit: 17 Feb 2025
Posts: 1,694
Own Kudos:
15,030
 [8]
Given Kudos: 766
Expert
Expert reply
Posts: 1,694
Kudos: 15,030
 [8]
1
Kudos
Add Kudos
7
Bookmarks
Bookmark this Post
General Discussion
User avatar
ramlala
Joined: 22 Aug 2020
Last visit: 13 Dec 2022
Posts: 470
Own Kudos:
Given Kudos: 30
Location: India
Concentration: International Business, Finance
GPA: 4
WE:Project Management (Energy)
Kudos
Add Kudos
Bookmarks
Bookmark this Post
avatar
Gknight5603
Joined: 26 Oct 2019
Last visit: 03 Apr 2022
Posts: 131
Own Kudos:
Given Kudos: 292
Location: India
GMAT 1: 680 Q49 V34
GPA: 4
GMAT 1: 680 Q49 V34
Posts: 131
Kudos: 55
Kudos
Add Kudos
Bookmarks
Bookmark this Post
ans D
if the plan is supprted by majority then by supporting the plan party and mayor may increase their chances in upcoming elections

Posted from my mobile device
User avatar
rishi02
Joined: 21 Sep 2015
Last visit: 06 Jan 2025
Posts: 86
Own Kudos:
500
 [1]
Given Kudos: 403
Location: India
GMAT 1: 730 Q48 V42
GMAT 2: 750 Q50 V41
GMAT 3: 760 Q49 V46
Products:
GMAT 3: 760 Q49 V46
Posts: 86
Kudos: 500
 [1]
1
Kudos
Add Kudos
Bookmarks
Bookmark this Post
EMPOWERgmatVerbal
Official Explanation:

Scott: In our town’s upcoming election, the mayor is running for re-election against an opponent whose popularity is currently greater than the mayor’s because of the opponent’s support for a plan that would forbid national chain stores in the downtown area. The mayor and his party should take note of this and pledge to support limiting such stores downtown.

The argument above relies on which of the following assumptions?

A. The mayor, by announcing a limit on national chain stores downtown, would increase voter turnout in the upcoming election.
B. Some voters who plan to support the mayor’s opponent would not do so if the mayor limits national chain stores downtown.
C. The mayor has the authority to limit the opening of national chain stores downtown.
D. The plan to forbid national chain stores downtown is supported by a majority of the town’s residents.
E. Most voters who currently plan to vote for the mayor’s opponent do not currently know of the mayor’s position on this issue.


Question Type: Assumption
Boil It Down: Because the opponent is more popular than the mayor due to the opponent’s position, the mayor should adopt the opponent’s position.
Goal: Use the negation test (explained below) to find the option that best reflects an assumption that MUST be made for the argument to work.

Analysis:

General comments about assumption questions:

Assumption style questions deal with a gap between the premise(s) and conclusion. The assumption will bridge that gap. For example, take the following argument:

P1: The Lakers are the best team in basketball
( )
C: Therefore, the Lakers will win the championship

There is a gap between the premise conclusion. This gap can be filled by identifying the assumption that the best team will win the championship. If this assumption is placed within the gap, the conclusion follows.

Assumption questions come in two main varieties:

1. Necessary assumption: Here, the assumption is required in order to move from premise(s) to conclusion. Without this assumption, the conclusion does not follow. Using the negation test can be a useful strategy with necessary assumptions. Deny the answer choice and see if the conclusion becomes impossible. If so, then that is the correct answer.
2. Sufficient assumption: Here, the assumption guarantees that the conclusion will occur. However, the conclusion may also occur without the presence of the assumption

This argument assumes that the mayor’s popularity will increase if he adopts his opponent’s position against national chains in the downtown area.

The question stem uses the phrase “relies on” which is another way of asking for a necessary assumption. For necessary assumption questions, we will do the negation test.

How to do the negation test?
Deny the answer choice and see if the conclusion becomes impossible. If so, then that is the correct answer. For example, consider the following argument:

P1: All puppies are young mammals.
P2: (Assumption )
C: All puppies are cute

Answer choices:
All puppies have round eyes
All young mammals are puppies
Some young mammals are cute

To do the negation test, we deny the answer choice and see if the conclusion is still possible. If the conclusion is still possible, then the answer choice is not necessary, and therefore, is not the correct answer.

If we deny A and say that it is not the case that all puppies have round eyes (or some puppies do not have round eyes), is it still possible that all puppies are cute? Yes. So, answer choice A is not correct. If we deny B and say that it is not the case that all young mammals are puppies (or some young mammals are not puppies), is it still possible that all puppies are cute? Yes. So, answer choice B is not correct. If we deny that some young mammals are cute by saying no young mammals are cute, is it still possible that all puppies are cute? No. Given that all puppies are young mammals, in order for all puppies to be cute, then it must be the case that some young mammals are cute. So, answer choice C is a necessary assumption for this argument.

Although I will use the negation test for evaluating the answer choices below, a more direct way of doing this question is to look to see if the answer choices are relevant. The argument connects the position on forbidding/limiting chains in the downtown area to the mayor’s increase in popularity. As with so many of these critical reasoning questions, seeing the connection between the premises and conclusions is key. Here is a quick breakdown of the argument at hand:

Argument structure:
(P) = Premise/Evidence/Support (C) = Conclusion

P1: Opponent is more popular than the mayor because of opponent’s position against national chains in the downtown
(P2: Assumption )

C: If the mayor wants to increase his popularity and win the election, the mayor and his party should support limiting national chain stores in the downtown area

Let’s see how each option holds up to the negation test and relevancy check:

A. The mayor, by announcing a limit on national chain stores downtown, would increase voter turnout in the upcoming election.
Incorrect. Suppose there was not an increase in voter turnout. The mayor could still win the election by becoming more popular than his opponent. His winning the election does not depend on voter turnout but on increasing his popularity. Therefore, answer choice A is not a necessary assumption.

B. Some voters who plan to support the mayor’s opponent would not do so if the mayor limits national chain stores downtown.
Correct. Suppose we denied this answer choice by saying that the mayor’s support for limiting national chains would not alter voter support for the opponent. Then, there is no way for the mayor to increase his popularity and win. By denying this answer choice, the conclusion becomes impossible. Therefore, answer choice B is a necessary assumption.

C. The mayor has the authority to limit the opening of national chain stores downtown.
Incorrect. Suppose the mayor did not have the authority to limit the opening of national chains in the downtown area. It is still possible that his adopting his opponent’s position could win voter’s over to the mayor’s side, regardless of his lack of authority to limit national chains. Therefore, answer choice C is not a necessary assumption.

D. The plan to forbid national chain stores downtown is supported by a majority of the town’s residents.
Incorrect. Suppose the plan to forbid national chain stores in the downtown is not supported by the majority of the town’s residents. The issue is increasing the mayor’s popularity relative to his opponent. If his opponent is more popular because of this one issue, then the mayor can still increase in popularity by changing his stance, even if the majority do not support this position. Therefore, answer choice D is not a necessary assumption.

E. Most voters who currently plan to vote for the mayor’s opponent do not currently know of the mayor’s position on this issue.
Incorrect. Suppose that most of the voters who plan on voting for the opponent did know the mayor’s position. The mayor could still increase in popularity by sniping some of the opponent’s supporters if he changes position. Therefore, answer choice E is not a necessary assumption.


Key Takeaways:
1. Use the negation test for necessary assumption question
2. Pay attention to relevance by noticing how the premises connect to the conclusion. In this question, only answer choice B properly connected popularity with limiting national chains in the downtown area


Don’t study for the GMAT. Train for it.

While evaluating option D, is the following train of thought correct EMPOWERgmatVerbal:

There could be three kinds of voters. 1) In favor 2) Undecided 3) Against

Consider a scenario where the plan to forbid national chain stores in the downtown is not supported by the majority of the town’s residents. It could mean the split among categories 1:2:3 is 49%:0%: 51% in which case it would be disadvantageous for the mayor to adopt the stance

OR

It could be that the split among the categories is something like 30%:50%:20% in which scenario, as per literal translation, majority of the the town’s residents do not support the plan to forbid national chain stores in the downtown, yet relatively speaking there are more voters in favor than against and therefore it would be advantageous for the mayor to pledge support for the plan.
User avatar
EMPOWERgmatVerbal
User avatar
EMPOWERgmat Instructor
Joined: 23 Feb 2015
Last visit: 17 Feb 2025
Posts: 1,694
Own Kudos:
15,030
 [1]
Given Kudos: 766
Expert
Expert reply
Posts: 1,694
Kudos: 15,030
 [1]
1
Kudos
Add Kudos
Bookmarks
Bookmark this Post
rishi02
EMPOWERgmatVerbal
Official Explanation:

Scott: In our town’s upcoming election, the mayor is running for re-election against an opponent whose popularity is currently greater than the mayor’s because of the opponent’s support for a plan that would forbid national chain stores in the downtown area. The mayor and his party should take note of this and pledge to support limiting such stores downtown.

The argument above relies on which of the following assumptions?

A. The mayor, by announcing a limit on national chain stores downtown, would increase voter turnout in the upcoming election.
B. Some voters who plan to support the mayor’s opponent would not do so if the mayor limits national chain stores downtown.
C. The mayor has the authority to limit the opening of national chain stores downtown.
D. The plan to forbid national chain stores downtown is supported by a majority of the town’s residents.
E. Most voters who currently plan to vote for the mayor’s opponent do not currently know of the mayor’s position on this issue.


Question Type: Assumption
Boil It Down: Because the opponent is more popular than the mayor due to the opponent’s position, the mayor should adopt the opponent’s position.
Goal: Use the negation test (explained below) to find the option that best reflects an assumption that MUST be made for the argument to work.

Analysis:

General comments about assumption questions:

Assumption style questions deal with a gap between the premise(s) and conclusion. The assumption will bridge that gap. For example, take the following argument:

P1: The Lakers are the best team in basketball
( )
C: Therefore, the Lakers will win the championship

There is a gap between the premise conclusion. This gap can be filled by identifying the assumption that the best team will win the championship. If this assumption is placed within the gap, the conclusion follows.

Assumption questions come in two main varieties:

1. Necessary assumption: Here, the assumption is required in order to move from premise(s) to conclusion. Without this assumption, the conclusion does not follow. Using the negation test can be a useful strategy with necessary assumptions. Deny the answer choice and see if the conclusion becomes impossible. If so, then that is the correct answer.
2. Sufficient assumption: Here, the assumption guarantees that the conclusion will occur. However, the conclusion may also occur without the presence of the assumption

This argument assumes that the mayor’s popularity will increase if he adopts his opponent’s position against national chains in the downtown area.

The question stem uses the phrase “relies on” which is another way of asking for a necessary assumption. For necessary assumption questions, we will do the negation test.

How to do the negation test?
Deny the answer choice and see if the conclusion becomes impossible. If so, then that is the correct answer. For example, consider the following argument:

P1: All puppies are young mammals.
P2: (Assumption )
C: All puppies are cute

Answer choices:
All puppies have round eyes
All young mammals are puppies
Some young mammals are cute

To do the negation test, we deny the answer choice and see if the conclusion is still possible. If the conclusion is still possible, then the answer choice is not necessary, and therefore, is not the correct answer.

If we deny A and say that it is not the case that all puppies have round eyes (or some puppies do not have round eyes), is it still possible that all puppies are cute? Yes. So, answer choice A is not correct. If we deny B and say that it is not the case that all young mammals are puppies (or some young mammals are not puppies), is it still possible that all puppies are cute? Yes. So, answer choice B is not correct. If we deny that some young mammals are cute by saying no young mammals are cute, is it still possible that all puppies are cute? No. Given that all puppies are young mammals, in order for all puppies to be cute, then it must be the case that some young mammals are cute. So, answer choice C is a necessary assumption for this argument.

Although I will use the negation test for evaluating the answer choices below, a more direct way of doing this question is to look to see if the answer choices are relevant. The argument connects the position on forbidding/limiting chains in the downtown area to the mayor’s increase in popularity. As with so many of these critical reasoning questions, seeing the connection between the premises and conclusions is key. Here is a quick breakdown of the argument at hand:

Argument structure:
(P) = Premise/Evidence/Support (C) = Conclusion

P1: Opponent is more popular than the mayor because of opponent’s position against national chains in the downtown
(P2: Assumption )

C: If the mayor wants to increase his popularity and win the election, the mayor and his party should support limiting national chain stores in the downtown area

Let’s see how each option holds up to the negation test and relevancy check:

A. The mayor, by announcing a limit on national chain stores downtown, would increase voter turnout in the upcoming election.
Incorrect. Suppose there was not an increase in voter turnout. The mayor could still win the election by becoming more popular than his opponent. His winning the election does not depend on voter turnout but on increasing his popularity. Therefore, answer choice A is not a necessary assumption.

B. Some voters who plan to support the mayor’s opponent would not do so if the mayor limits national chain stores downtown.
Correct. Suppose we denied this answer choice by saying that the mayor’s support for limiting national chains would not alter voter support for the opponent. Then, there is no way for the mayor to increase his popularity and win. By denying this answer choice, the conclusion becomes impossible. Therefore, answer choice B is a necessary assumption.

C. The mayor has the authority to limit the opening of national chain stores downtown.
Incorrect. Suppose the mayor did not have the authority to limit the opening of national chains in the downtown area. It is still possible that his adopting his opponent’s position could win voter’s over to the mayor’s side, regardless of his lack of authority to limit national chains. Therefore, answer choice C is not a necessary assumption.

D. The plan to forbid national chain stores downtown is supported by a majority of the town’s residents.
Incorrect. Suppose the plan to forbid national chain stores in the downtown is not supported by the majority of the town’s residents. The issue is increasing the mayor’s popularity relative to his opponent. If his opponent is more popular because of this one issue, then the mayor can still increase in popularity by changing his stance, even if the majority do not support this position. Therefore, answer choice D is not a necessary assumption.

E. Most voters who currently plan to vote for the mayor’s opponent do not currently know of the mayor’s position on this issue.
Incorrect. Suppose that most of the voters who plan on voting for the opponent did know the mayor’s position. The mayor could still increase in popularity by sniping some of the opponent’s supporters if he changes position. Therefore, answer choice E is not a necessary assumption.


Key Takeaways:
1. Use the negation test for necessary assumption question
2. Pay attention to relevance by noticing how the premises connect to the conclusion. In this question, only answer choice B properly connected popularity with limiting national chains in the downtown area


Don’t study for the GMAT. Train for it.

While evaluating option D, is the following train of thought correct EMPOWERgmatVerbal:

There could be three kinds of voters. 1) In favor 2) Undecided 3) Against

Consider a scenario where the plan to forbid national chain stores in the downtown is not supported by the majority of the town’s residents. It could mean the split among categories 1:2:3 is 49%:0%: 51% in which case it would be disadvantageous for the mayor to adopt the stance

OR

It could be that the split among the categories is something like 30%:50%:20% in which scenario, as per literal translation, majority of the the town’s residents do not support the plan to forbid national chain stores in the downtown, yet relatively speaking there are more voters in favor than against and therefore it would be advantageous for the mayor to pledge support for the plan.

Great question rishi02!

The main reason option D doesn't work is that the assumption is a step too far away from the most basic assumption we need to make for this argument to even work: will voters actually change who they vote for if the mayor changes his stance? If we can assume that yes, voters will change their votes if the mayor changes his stance, then it'll work. If we assume that voters won't change who they vote for, then the plan is pointless. THIS is the most important piece of information we would need to know to determine if this plan would work.

Whether or not the majority of people in town support the issue is important, but we FIRST need to determine if voters will even change their votes in the first place. We see this a lot with voters who vote only with their party - even if the other candidate agrees with all of their opinions/wants/needs as a voter, some voters still vote the way they always do. However, if we can show that at least SOME voters will switch candidates based on this, then the plan is likely going to work.

I hope that helps! Keep tagging us at EMPOWERgmatVerbal if you have any other questions!
User avatar
VerbalBot
User avatar
Non-Human User
Joined: 01 Oct 2013
Last visit: 04 Jan 2021
Posts: 18,396
Own Kudos:
Posts: 18,396
Kudos: 946
Kudos
Add Kudos
Bookmarks
Bookmark this Post
Hello from the GMAT Club VerbalBot!

Thanks to another GMAT Club member, I have just discovered this valuable topic, yet it had no discussion for over a year. I am now bumping it up - doing my job. I think you may find it valuable (esp those replies with Kudos).

Want to see all other topics I dig out? Follow me (click follow button on profile). You will receive a summary of all topics I bump in your profile area as well as via email.
Moderators:
GMAT Club Verbal Expert
7332 posts
GMAT Club Verbal Expert
235 posts